Anda di halaman 1dari 25

Abstract Algebra

Chapter 13 - Field Theory


David S. Dummit and Richard M. Foote
Solution Manual by positrn0802

Contents

13 Field Theory 1
13.1 Basic Theory and Field Extensions . . . . . . . . . . . . . . . . . . . . . . . . . 1
13.2 Algebraic Extensions . . . . . . . . . . . . . . . . . . . . . . . . . . . . . . . . . 4
13.3 Classical Straightedge and Compass Constructions . . . . . . . . . . . . . . . . 12
13.4 Splitting Fields and Algebraic Closures . . . . . . . . . . . . . . . . . . . . . . . 13
13.5 Separable and Inseparable Extension . . . . . . . . . . . . . . . . . . . . . . . . 15
13.6 Cyclotomic Polynomials and Extensions . . . . . . . . . . . . . . . . . . . . . . 19

13 Field Theory

13.1 Basic Theory and Field Extensions


Exercise 1. Show that p(x) = x3 + 9x + 6 is irreducible in Q[x]. Let be a root of p(x).
Find the inverse of 1 + Q()
Solution: p(x) = x3 + 9x + 6 is irreducible in Z[x] by Eisenstein Criterion with p = 3.
By Gauss Lemma, then it is irreducible in Q[x]. To find (1 + )1 , we apply the Euclidean
algorithm (long division) to p(x) and 1 + x. We find

x3 + 9x + 6 = (1 + x)(x2 x + 10) 4.

Evaluating at , we find (1 + )(2 + 10) = 4. Therefore

2 + 10
(1 + )1 = .
4

Exercise 2. Show that x3 2x 2 is irreducible over Q and let be a root. Compute


1+
(1 + )(1 + + 2 ) and in Q().
1 + + 2
Solution: Let f (x) = x3 2x 2. f is irreducible over Z by Eisenstein Criterion with
p = 2, hence over Q by Gauss Lemma. Now, if is a root of f , then 3 = 2 + 2. Hence

(1 + )(1 + + 2 ) = 1 + 2 + 22 + 3 = 3 + 4 + 22 .
1+
For computing , first we compute (1 + + 2 )1 . Applying the Euclidean algorithm,
1 + + 2
we obtain
x3 2x 2 = (x2 + x + 1)(x 1) 2x 1,
and
x2 x 7 9
x3 2x 2 = (2x + 1)( ) .
2 4 8 8

1
13.1 Basic Theory and Field Extensions

Evaluating at , from this equalities we obtain

8 2 7
(2 + + 1)( 1) = 2 + 1 and (2 + 1)1 = ( ).
9 2 4 8
Combining these two equations we obtain

8 2 2 7
( + + 1)( 1)( ) = 1.
9 2 4 8
So,
8 2 7 22 5
(2 + + 1)1 = ( 1)( ) = + + ,
9 2 4 8 3 3 3
3
where we used = 2 + 2 again. Therefore,

1+ 22 5 2 2 1
= (1 + )( + + ) = + + .
1 + + 2 3 3 3 3 3 3

Exercise 3. Show that x3 + x + 1 is irreducible over F2 and let be a root. Compute the
powers of in F2 ().
Solution: Since 03 + 0 + 1 = 1 and 11 + 1 + 1 = 1 in F2 , then x3 + x + 1 is irreducible over
F2 . Since is root of x3 + x + 1, then 3 = 1 = + 1. Hence, the powers of in F2 () are

, 2 , 3 = + 1, 4 = 2 + , 5 = 2 + + 1, 6 = 2 + 1, and 7 = 1.

Exercise 4. Prove directly that the map a + b 2 a b 2 is an isomorphism of Q( 2)
with itself.
Solution: Denote this map by . Then

(a + b 2 + c + d 2) = a + c b 2 d 2 = (a + b 2) + (c + d 2),

and
((a + b 2) (c + d 2)) = (ac + 2bd + (ad + bc) 2)

= ac + 2bd (ad + bc) 2

= (a b 2)(c d 2)

= (a + b 2)(c + d 2),

hence is anhomomorphism. Moreover, if (a + b 2) = (c + d 2), then a b 2 =c d 2,
hence (since
2 6 Q( 2), then
Q) a = b and c = d, so is injective. Also, given a + b 2
(a b 2) = a + b 2, so is surjective. Therefore, is an isomorphism of Q( 2) with itself.
Exercise 5. Suppose is a rational root of a monic polynomial in Z[x]. Prove that is
an integer.
Solution: Let = p/q be a root of a monic polynomial p(x) = xn + + a1 x + a0 over Z,
with gcd(p, q) = 1. Then
p p p
( )n + an1 ( )n1 + + a1 + a0 = 0.
q q q
Multiplying this equation by q n one obtains

pn + an1 pn1 q + + a1 pq n1 + a0 q n = 0
q(an1 pn1 + + a1 pq n2 + a0 q n1 ) = pn .

2
13.1 Basic Theory and Field Extensions

Thus, every prime that divides q divides pn as well, so divides p. Since gcd(p, q) = 1, there is
no prime dividing q, hence q = 1. The result follows.
Exercise 6. Show that if is a root of an xn + an1 xn1 + + a1 x + a0 then an is a
root of the monic polynomial xn + an1 xn1 + an an2 xn2 + + an2 n1
n a1 x + an a0 .
Solution: This is straightforward. If

an n + an1 n1 + + a1 + a0 = 0,

then
(an )n + an1 (an )n1 + an an2 (an )n2 + + ann2 a1 (an ) + ann1 a0
= ann n + an1
n an1
n1
+ ann1 an2 n2 + + an1 n1
n a1 + an a0
= an1 n
n (an + an1
n1
+ an2 n2 + + a1 + a0 ) = 0.

Exercise 7. Prove that x3 nx + 2 is irreducible for n 6= 1, 3, 5.


Solution: If x3 nx + 2 is reducible it must have a linear factor, hence a root. By Rational
Root Theorem, if is a root of x3 nx + 2, then must divide its constant term, so the
possibilities are = 1, 2. If = 1 or 2, then n = 3; if = 1, then n = 1; and if = 2,
then n = 5. Therefore, x3 nx + 2 is irreducible for n 6= 1, 3, 5.
Exercise 8. Prove that x5 ax 1 Z[x] is irreducible unless a = 0, 2 or 1. The first two
correspond to linear factors, the third corresponds to the factorization (x2 x + 1)(x3 + x2 1).
Solution: We subdivide this exercise in cases and subcases.
If x5 ax 1 is reducible then it has a root (linear factor) or is a product of two irreducible
polynomials of degrees 2 and 3.
Case 1. If x5 ax 1 has a root, then, by Rational Root Theorem, it must be = 1. If
= 1 is a root, then a = 0. If = 1 is a root, then a = 2.
Case 2. Now, suppose that there exists f (x) and g(x) irreducible monic polynomials over
Z of degrees 2 and 3 respectively, such that x5 ax 1 = f (x)g(x). Write f (x) = x2 + bx + c
and g(x) = x3 + rx2 + sx + t, where b, c, r, s, t Z. Then

x5 ax 1 = (x2 + bx + c)(x3 + rx2 + sx + t)


= x5 + (b + r)x4 + (br + c + s)x3 + (bs + cr + t)x2 + (bt + cs) + tc.

Equating coefficients leads to


b+r =0
br + c + s = 0
bs + cr + t = 0
bt + cs = a
ct = 1.
From ct = 1 we deduce (c, t) = (1, 1) of (c, t) = (1, 1), which give us two cases.
Case 2.1. First suppose (c, t) = (1, 1). Then the system of equations reduces to

b+r =0
br 1 + s = 0
bs r + 1 = 0
b s = a.

Now, put b = r into second and third equations to obtain r2 1 + s = 0 and rs r + 1 = 0,


that is, r2 + 1 s = 0 and rs + r 1 = 0. Adding these last two equations we obtain

3
13.2 Algebraic Extensions

r2 + rs + r s = 0. Thus r2 + rs + r + s = 2s, so (r + 1)(r + s) = 2s. Now, from r2 + 1 s = 0


we have r2 = s 1, so then r2 + rs + r s = 0 becomes rs + r = 1, that is, r(s + 1) = 1. Hence,
r = 1 and s = 0, or r = 1 and s = 2. If r = 1 and s = 0, then (r + 1)(r + s) = 2s leads to
2 = 0, a contradiction. If r = 1 and s = 2, it leads to 0 = 4, another contradiction.
Therefore, (c, t) = (1, 1) is impossible. We now pass to the case (c, t) = (1, 1).
Case 2.2. Suppose that (c, t) = (1, 1). The system of equations reduces to

b+r =0
br + 1 + s = 0
bs + r 1 = 0
b + s = a.

Adding the second and third equation we obtain b(r + s) + r + s = 0, so that (b + 1)(r + s) = 0.
Then b = 1 or r = s, so one more time we have two cases. If r = s, then br + 1 + s = 0
becomes br + 1 r = 0. Hence, b = r and br + 1 r = 0 gives r2 + r 1 = 0. By Rational
Root Theorem, this equation has no roots on Z. Since r Z, we have a contradiction. Now
suppose b = 1. From b = r we obtain r = 1, so, from br + 1 + s = 0 we obtain s = 0. Finally,
from b + s = a we obtain a = 1. Therefore, the solution (b, c, r, s, t) = (1, 1, 1, 0, 1) is
consistent and we obtain the factorization

x5 ax 1 = (x2 + bx + c)(x3 + rx2 + sx + t) = (x2 x + 1)(x3 + x2 1).

13.2 Algebraic Extensions


Exercise 1. Let F be a finite field of characteristic p. Prove that |F| = pn for some positive
integer n.
Solution: Since the characteristic of F is p, its prime subfield is (isomorphic to) Fp = Z/pZ.
We can consider F as a vector space over Fp . Since F is finite, then [F : Fp ] = n for some n Z+ .
Therefore
|F| = |Fp |[F:Fp ] = pn .

Exercise 2. Let g(x) = x2 + x 1 and let h(x) = x3 x + 1. Obtain fields of 4, 8, 9 and


27 elements by adjoining a root of f (x) to the field F where f (x) = g(x) of h(x) and F = F2
of F3 . Write down the multiplication tables for the field with 4 and 9 elements and show that
the nonzero elements form a cyclic group.
Solution: Note that g and h are irreducible over F2 and F3 . Now, is is a root of g, then
F2 ()
= F2 /(g(x)) has 4 elements and F3 () = F3 /(g(x)) has 9 elements. Furthermore, is 2 is
a root of h, then F2 (2 )
= F2 /(h(x)) has 8 elements and F3 (2 )
= F3 /(h(x)) has 27 elements.
The multiplication table for F2 /(g(x)) is

0 1 x x+1
0 0 0 0 0
1 0 1 x x+1
x 0 x x+1 x
x+1 0 x+1 x x

The multiplication table for F3 /(g(x)) is

4
13.2 Algebraic Extensions

0 1 2 x x+1 x+2 2x 2x + 1 2x + 2
0 0 0 0 0 0 0 0 0 0
1 0 1 2 x x+1 x+2 2x 2x + 1 2x + 2
2 0 2 1 2x 2x + 2 2x + 1 x x+2 x+1
x 0 x 2x 2x + 1 1 x+1 x+2 2x + 2 2
x+1 0 x+1 2x + 2 1 x+2 2x 2 x 2x + 1
x+2 0 x+2 2x + 1 x+1 2x 2 2x + 2 1 x
2x 0 2x x x+2 2 2x + 2 2x + 1 x+1 1
2x + 1 0 2x + 1 x+2 2x + 2 x 1 x+1 2 2x
2x + 2 0 2x + 2 x+1 2 2x + 1 x 1 2x x+2

In both cases, x is a generator of the cyclic group of nonzero elements.


Exercise 3. Determine the minimal polynomial over Q for the element 1 + i.
Solution: Since 1+i 6 Q, its minimal polynomial is of degree at least 2. We try conjugation,
and obtain
(x (1 + i))(x (1 i)) = x2 2x + 2,
which is irreducible by Eisenstein with p = 2. Therefore, the minimal polynomial is x2 2x + 2.

Exercise 4. Determine the degree over Q of 2 + 3 and 1 + 3 2 + 3 4.

Solution: First,
note that
(2 + 3)2 = 4 + 4 3 + 3 = 7 + 4 3. Let = 2 + 3. Then
2 4 = 7 + 4 3 8 4 3 = 1, hence is a root of x2 4x + 1. Moreover, x2 4x +1
is irreducible over Q (because
6 Q), so x 2 4x + 1 is the minimal polynomial of 2 + 3.

Therefore, 2 + 3 has degree 2 over Q.
Now, let = 3 2 and = 1 + + 2 . Then Q(), so Q Q() Q(). We have
[Q() : Q] = [Q() : Q()][Q() : Q]. Note that [Q() : Q] = 3 since has minimal polynomial
x3 2 over Q, so [Q() : Q] = 1 or 3. For a contradiction, suppose [Q() : Q] = 1, that is,
Q() = Q so Q. Then

2 = (1 + + 2 )2 = 1 + 2 + 32 + 23 + 4 = 5 + 4 + 32 ,

where we used 3 = 2. So

2 3 = 5 + 2 + 32 3(1 + + 2 ) = 2 ,

hence = 2 + 3 + 2 Q() = Q, a contradiction. Therefore, [Q() : Q] = 3.


Exercise 5. Let F = Q(i). Prove that x3 2 and x3 3 are irreducible over F .
Solution: Since the polynomials have degree 3, if they were reducible they must have a
linear factor, hence a root in F .Note element of F is of the form a + bi, where
that every
a, b Q. The roots of x3 2 are 3 2, 3 2 and 2 3 2, where is the primitive 3rd root of unity,
1

3

i.e., = exp(2i/3) = cos(2/3) + i sin(2/3) = 2 + 2 . Since 3 6 Q, none of this elements

is in F , hence x3 2 is irreducible over F . Similarly, the roots of x3 3 are 3 3, 3 3 and 2 3 3,
and by the same argument non of this elements is in F . Hence x3 3 is irreducible over F .
Exercise 6. Prove directly from the definitions that the field F (1 , 2 , . . . , n ) is the
composite of the fields F (1 ), F (2 ), . . . , F (n ).
Solution: We have to prove that F (1 , . . . , n ) is the smallest field containing F (1 ), . . . , F (n ).
Clearly F (i ) F (1 , . . . , n ) for all 1 i n. Now let K be a field such that F (i ) K
for all i. If is an element of F (1 , . . . , n ), then is of the form = a1 1 + + an n ,
where a1 , . . . , an F . Every ai i is in K, hence K. Thus F (1 , . . . , n ) K. Therefore,

5
13.2 Algebraic Extensions

F (1 , . . . , n ) contains all F (i ) and is contained in every field containing all F (i ), hence


F (1 , . . . , n ) is the composite of the fields F (1 ), F (2 ), . . . , F (n ).

Exercise 7. Prove that Q( 2 + 3) = Q( 2, 3) [one inclusion is obvious, for the other
consider ( 2+ 3)2 , etc.]. Conclude that [Q( 2+ 3) : Q] = 4. Find an irreducible polynomial

satisfied by 2 + 3.

Solution: Since 2 + 3 is in Q( 2,
3), clearly Q(
2 + 3) Q( 2, 3).
For the other
direction we have to prove that 2 and 2 are in Q( 2 + 3). Let = 2 + 3. Then

2 = 5 + 2 6, 3 = 11 2 + 9 3 and 4 = 37 + 15 6.

So
1 1
2 = (3 9), 3 = (11 3 ) and 4 = 49 + 20 6.
2 2

Therefore 2 Q() and 3 Q(),
so Q( 2, 3) Q( 2 + 3). The equality follows.
Hence, [Q( 2 + 3) : Q] = [Q( 2, 3) : Q] = 4.
We also have

4 102 = (49 + 20 6) 10(5 + 2 6) = 1, so 4 102 + 1 = 0.

Since [Q( 2+ 3) : Q] = 4, then x4 10x2 +1 is irreducible over Q, and is satisfied by 2+ 3.
Exercise 8. Let F be a field of characteristic
6= 2. Let D1 and D2 be elements of F , neither
of which is a square in F . Prove that F ( D1 , D2 ) is of 4 over F if D1 D2 is not a
degree
square in F and is of degree 2 over F otherwise. When F ( D1 , D2 ) is of degree 4 over F the
field is called a biquadratic extension of F .

Solution: The elements of F ( D1 , D2 ) can be written in the form
p p p
a + b D1 + c D2 + d D1 D2 , where a, b, c, d F.

We have p p p p p p
[F ( D1 , D2 ) : F ] = [F ( D1 , D2 ) : F ( D1 )][F ( D1 ) : F ].

Since [F ( D1 ) : F ] = 2,
then [F (
D 1 , D 2 ) : F ] can be 2 or 4. Now, [F ( D 1 , D2 ) : F ] = 2
if andonly if [F ( D1 , D2 ) : F ( D1 )] = 1, and that occurs exactly when x2 D2 is reducible
in F ( D1 ) (i.e., when D2 F ( D1 )), that is, if there exists a, b F such that
p p
(a + b D1 )2 = D2 , so that a2 + 2ab D1 + b2 D12 = D2 .

Note that ab = 0 as ab 6= 0 implies D1 F , contrary to the hypothesis. Then a = 0 or b = 0.
If b = 0, then D2 is a square in F , contrary to the hypothesis. If a = 0, thenb2 D1 = D2 , and
thus D1 D2 = ( Db2 )2 , so D1 D2 is a square in F . So, x2 D2 is reducible in F ( D1 ) if and only
if D1 D2 is a square in F . The result follows.
Exercise 9. Let F be a field of characteristic 6= 2. Let a, b be elements
p of the field F with

b not a square in F . Prove that a necessary and sufficient condition for a + b = m + n
forpsome m and n in F is that a2 b is a square in F . Use this to determine when the field

Q( a + b)(a, b Q) is biquadratic over Q.
p
Solution: Suppose a + b = m+ n for some m, n F , then a+ b = m+n+2 mn.
p
Since b is not a square in F , this means b = 2 mn. We also have a + b n = m, so

q
b = 2 n( a + b n).

6
13.2 Algebraic Extensions

Hence,
q
b = 2 n(a + b) 2n

( b + 2n)2 = 4n(a + b)

b + 4n b + 4n2 = 4n(a + b)
b + 4n2 4na = 0

4a 16a2 16b
n=
8
p 2n
a2 b = .
a

Therefore, since a and n are in F , a2 b is in F .
2 2
Now suppose that p a b is asquarein F , so that a b F . We prove that there exists
m, n F such that a + b = m + n. Let

a + a2 b a a2 b
m= and n = .
2 2
p
Note that m and n are in F as char(F ) 6= 2. We claim a + b = m + n. Indeed, we have
p p
(a + b) + 2 a2 b + (a b) a+ b+ a b 2
m= =( ) ,
4 2
and p p
(a + b) 2 a2 b + (a b) a+ b a b 2
n= =( ) .
4 2
Thus p p p p
a+ b+ a b a+ b a b
m= and n= .
2 2
Therefore,
p p p p

q
a+ b+ a b a+ b a b
m+ n= + = a + b,
2 2
as claimed. p
Now, we this to determine when the field Q(pa + b), a, b Q, is biquadratic over Q. If

a2 b is a square in
p Q and b is not, we have Q( a + b) = Q( m + n) = Q( m, n), so

by last exercise Q( a + b) is biquadratic over Q when a2 b is a square in Q, and neither b,
m, n or mn are squares in Q. Since

a + a2 b a a2 b b
mn = = ,
2 2 4
p
then mn is never a square when b isnt. Thus, Q( a + b) is biquadratic over Q exactly when
a2 b is a square in Q and neither b, m nor n is a square in Q.
p
Exercise 10. Determine the degree of the extension Q( 3 + 2 2) over Q.
p p
Solution: Note that 3 + 2 2 = 3 + 8. Recalling last exercise with a = 3 and b = 8,
we have a2 b = 9 p8 = 1is a square in Q and b = 8 ispnot. Hence, we find (m = 2 and n = 1

p 3 + 8 = 2 + 1. Therefore, Q( 3 + 2 2) = Q( 2) and the degree of
from last exercise)
the extension Q( 3 + 2 2) over Q is 2.

7
13.2 Algebraic Extensions


Exercise 11. (a) Let 3 + 4i denote the square root of the complex number 3 + 4i that
let 3
lies in the first quadrant and 4i denote the square root of 3 4i that lies in the fourth
quadrant. Prove that [Q( 3 + 4i + 3 4i) : Q] p = 1.
p
(b) Determine the degree of the extension Q( 1 + 3 + 1 3).
Solution: (a) First, note that the conjugation map a + bi a bi is an isomorphism of C,
so it takes squares roots to
square roots, and maps numbers of the first quadrant to the fourth
(and reciprocally). Since 3 + 4i is the square root of 3 + 4i in the first quadrant,
its conjugate

is the square of root of 3 4i in the fourth quadrant, so is 3 4i. Hence 3 + 4ip and 3 4i
are conjugates each other. Now, we use Exercise 9 again. Note that 3 + 4i = 3 + 16.
With a = 3 and b = 16, we have a2 b = 25 is a square in Q and b = 16 is not. Hence,
we
find m = 1 and n = 4 and thus 3 + 4i = 1 + 4
= 1 + 2i. Furthermore, we find
3 4i = 1 2i.
pTherefore,
p3 + 4i+ 3 4i = 4, i.e., 3 + 4i + 3 4i Q.
(b) Let = 1 + 3 + 1 3. Then

q q
= ( 1 + 3 + 1 3)2 = (1 + 3) + (2 1 + 3) + (1 3) = 6.
2

Since x2 6 is irreducible over Q (Eisenstein p = 2), then has degree 2 over Q.


Exercise 12. Suppose the degree of the extension K/F is a prime p. Show that any subfield
E of K containing F is either K or F .
Solution: Let E be a subfield of K containing F . Then

[K : F ] = [K : E][E : F ] = p.

Since p is prime, either [K : E] = 1 or [E : F ] = 1. The result follows.

Exercise 13. Suppose F = Q(1 , 2 , . . . , n ) where i2 Q for i = 1, 2, . . . , n. Prove that


3
2 6 F .
Solution: Note that, for all 1 k n, we have [Q(1 , . . . , k ) : Q(
1 , . . . , k1 )] =
1 or 2.
m 3
Then [F : Q] = 2 for some m N. Suppose 2 F . Then Q 3 3
Q( 2) F , so [Q( 2) : Q]
divides [F : Q], that is, 3 divides 2m , a contradiction. Hence, 3 2 6 F .
Exercise 14. Prove that if [F () : F ] is odd then F () = F (2 ).
Solution: Since 2 F (), clearly F (2 ) F (). Thus we have to prove F (2 ). For
this purpose, consider the polynomial p(x) = x2 2 , so that p() = 0. Note that F (2 ) if
and only if p(x) is reducible in F (2 ). For a contradiction, suppose p(x) is irreducible in F (2 ),
so that [F () : F (2 )] = 2. Thus

[F () : F ] = [F () : F (2 )][F (2 ) : F ] = 2[F (2 ) : F ],

so [F () : F ] is even, a contradiction. Therefore, p(x) is reducible in F (2 ) and F (2 ).


Exercise 15. A field F is said to be formally real if 1 is not expressible as a sum of
squares in F . Let F be a formally real field, let f (x) F [x] be an irreducible polynomial
of odd degree and let be a root of f (x). Prove that F () is also formally real. [Pick a
counterexample of minimal degree. Show that 1 + f (x)g(x) = (p1 (x))2 + + (pm (x))2 for
some pi (x), g(x) F [x] where g(x) has odd degree < deg f . Show that some root of g has
odd degree over F and F () is not formally real, violating the minimality of .]
Solution: We follow the hint. Suppose there exists a counterexample. Let be of minimal
degree such that F () is not formally real and having minimal polynomial f of odd degree,
say deg f = 2k + 1 for some k N. Since F () is not formally real, then 1 can be express as

8
13.2 Algebraic Extensions

a sum of squares in F ()
= F [x]/((f (x))). Then, the exists polynomials p1 (x), . . . , pm (x), g(x)
such that
1 + f (x)g(x) = (p1 (x))2 + + (pm (x))2 .
As every element in F [x]/((f (x)) can be written as a polynomial in with degree less than
deg f , we have deg pi < 2k + 1 for all i. Thus, the degree in the right hand of the equation is
less than 4k + 1, so deg g < 2k + 1 as well. We prove that the degree of g is odd by proving
that the degree of (p1 (x))2 + + (pm (x))2 is even, because then the equation 1 + f (x)g(x) =
(p1 (x))2 + + (pm (x))2 implies the result. Let d be the maximal degree over all pi , we prove
that x2d is the leading term of (p1 (x))2 + + (pm (x))2 . Note that x2d is a sum of squares (of
the leading coefficients of the pi s of maximal P degree). Now, sincePF is formally real, 0 cant be
expressed as a sum of squares in F . Indeed, if li=1 a2i = 0, then l1 2
i=1 (ai /al ) = 1. Therefore
2d 2 2
x 6= 0, so the degree of (p1 (x)) + + (pm (x)) is 2d, as claimed. Hence, the degree of g
must be odd by the assertion above. Then g must contain an irreducible factor of odd degree,
say h(x). Since deg g < deg f , we have deg h < deg f as well. Let be a root of h(x), hence a
root of g(x). Then
f (x)g(x)
1 + h(x) = (p1 (x))2 + + (pm (x))2 ,
h(x)
so 1 is a square in F [x]/((h(x))
= F (), which means F () is not formally real. Therefore,
is a root of an odd degree polynomial h such that F () is not formally real. Since deg h <
deg f , this contradicts the minimality of . The result follows.
Exercise 16. Let K/F be an algebraic extension and let R be a ring contained in K and
containing F . Show that R is a subfield of K containing F .
Solution: Let r R be nonzero. Since r is algebraic over F , there exist an irreducible
polynomial p(x) = a0 + a1 x + + xn F [x] such that p(r) = 0. Note that a0 6= 0 since p is
irreducible. Then r1 = a1
0 (r
n1 + + a ). Since a F R and r R, we have r 1 R.
1 i

Exercise 17. Let f (x) be an irreducible polynomial of degree n over a field F . Let g(x)
be any polynomial in F [x]. Prove that every irreducible factor of the composite polynomial
f (g(x)) has degree divisible by n.
Solution: Let p(x) be an irreducible factor of f (g(x)) of degree m. Let be a root of p(x).
Since p is irreducible, then [F () : F ] = deg p(x) = m. Now, since p(x) divides f (g(x)), we have
f (g()) = 0 and thus g() is a root of f (x). Since f is irreducible, this means n = [F (g()) : F ].
Note that F (g()) F (). Therefore,

n = [F (g()) : F ] = [F (g()) : F ()][F () : F ] = [F (g()) : F ()] m,

so m divides n, that is, deg p divides deg f .


Exercise 18. Let k be a field and let k(x) be the field of rational functions in x with
P (x)
coefficients from k. Let t k(x) be the rational function with relatively prime polynomials
Q(x)
P (x), Q(x) k[x], with Q(x) 6= 0. Then k(x) is an extension of k(t) and to compute its degree
it is necessary to compute the minimal polynomial with coefficients in k(t) satisfied by x.
(a) Show that the polynomial P (X) tQ(X) in the variable X and coefficients in k(t) is
irreducible over k(t) and has x as a root. [By Gauss Lemma this polynomial is irreducible in
(k(t))[X] if and only if it is irreducible in (k[t])[X]. Then note that (k[t])[X] = (k[X])[t].]
(b) Show that the degree of P (X) tQ(X) as a polynomial in X with coefficient in k(t) is
the maximum of the degree of P (x) and Q(x).
P (x)
(c) Show that [k(x) : k(t)] = [k(x) : k( )] = max(degP (x), degQ(x)).
Q(x)

9
13.2 Algebraic Extensions

Solution: (a) We follow the hint. Since k[t] is an UFD and k(t) is its field of fractions,
then, by Gauss Lemma, P (X) tQ(X) is irreducible in k((t))[X] is and only if it is irreducible
in (k[t])[X]. Note that (k[t])[X] = (k[X])[t]. Since P (X)tQ(X) is linear in (k[X])[t], is clearly
irreducible in (k[X])[t] (i.e., in (k[t])[X]), hence in (k(t))[X]. Thus, P (X)tQ(X) is irreducible
P (x)
in k(t). Now, x is clearly a root of P (X) tQ(X) since P (x) tQ(x) = P (x) Q(x) =
Q(x)
P (x) P (x) = 0.
(b) Let n = max{degP (x), degQ(x)}. Write

P (x) = an xn + + a1 x + a0 and Q(x) = bn xn + + b1 x + b0 ,

where ai , bi k for all i, so at least one of an or bn is nonzero. The degree of P (X) tQ(X) is
clearly n, we prove is n. If an or bn is zero then clearly deg (P (X) tQ(X)) = n. Suppose
an , bn 6= 0. Then an , bn k, but t 6 k (as t k(x)), it cannot be that an = tbn . Thus
(an tbn )X n 6= 0, so the degree of P (X) tQ(X) is n.
(c) Since P (X) tQ(X) is irreducible over k(t) and x is a root by part (a), then [k(x) :
k(t)] = degP (X) tQ(X), and this degree equals max{degP (x), degQ(x)} by part (b).
Exercise 19. Let K be an extension of F of degree n.
(a) For any K prove that acting by left multiplication on K is an F -linear transfor-
mation of K.
(b) Prove that K is isomorphic to a subfield of the ring of n n matrices over F , so the
ring of n n matrices over F contains an isomorphic copy of every extension of F of degree
n.
Solution: (a) Fix in K. Since K is (in particular) a commutative ring, we have (a+b) =
a + b and (a) = (a) for all a, b, K. If, in particular, F , we have the result.
(b) Fix a basis for K as a vector space over F . By part (a), for every K we can associate
a F -linear transformation T . Denote by T the matrix of T with respect to the basis fixed
above. Then define : K Mn (F ) by () = T . We claim is an isomorphism. Indeed,
if , K, then T(+) (k) = ( + )(k) = k + k = T (k) + T (k) for every k K, hence
T(+) = T + T . We also have T() (k) = ()(k) = kk = T (k)T (k) for every k K, so
T() = T T . Thus ( + ) = () + () and () = ()() (since the basis is fixed),
so is an homomorphism. Now, if () = (), then k = k for every k K, so letting
k = 1 we find that is injective. Therefore, (K) is isomorphic to a subfield of Mn (F ), so the
ring Mn (F ) contains an isomorphic copy of every extension of F of degree n.
Exercise 20. Show that if the matrix of the linear transformation "multiplication by "
considered in the previous exercise is A then is a root of the characteristic polynomial for A.
This gives an effective procedure for determining an equation of degree n satisfies by a element
in an extension
of F of degree
n. Use
this procedure to obtain the monic polynomial of degree
3 satisfied by 3 2 and by 1 + 3 2 + 3 4.
Solution: The characteristic polynomial of A is p(x) = det(Ix A). For every k K, we
have (I A)k = k Ak = k k = 0, so det(I
A) = 0 in K. Therefore, p() = 0.

Now, consider the field Q( 3 2) with
basis {1, 3
2, 3
4} over Q.
Denote
the elements of this
basis by e1 = 1, e2 = 3 2 and e3 = 3 4. Let = 3 2 and = 1 + 3 2 + 3 4. Then (e1 ) = e2 ,
(e2 ) = e3 and (e3 ) = 2e1 . We also have (e1 ) = e1 + e2 + e3 , (e2 ) = 2e1 + e2 + e3 and
(e3 ) = 2e1 + 2e2 + e3 . Thus, the associated matrices of the their linear transformations are,
respectively,
0 0 2 1 2 2
A = 1 0 0 and A = 1 1 2 .
0 1 0 1 1 1

10
13.2 Algebraic Extensions

The characteristic
polynomial of A is x3 2, hence is the monic polynomial of degree 3 satisfied
3
by = 2. Furthermore, the characteristic polynomial ofA is x3 3x2 3x 1, hence is the
monic polynomial of degree 3 satisfied by = 1 + 2 + 3 4.
3


Exercise 21. Let K = Q( D) for some squarefree integer D. Let = a + b D be an
element of K. Use the basis 1, D for K as a vector space over Q and show that the matrix of
the linear transformation "multiplication by " on K considered  in theprevious exercises has

 
a bD a bD
the matrix . Prove directly that the map a + b D 7 is an isomorphism of
b a b a
the field K with a subfield of the ring of 2 2 matrices with coefficients in Q.
Solution: The matrixof the linear transformation "multiplication
by " on K is found by
acting of in 
the basis 1, D. We have (1) = = a + b D and ( D) = a D
 + bD. Hence


a bD a bD
the matrix is . Now let : K M2 (Q) be defined by (a + b D) = .
b a b a
We have

     
a + c (b + d)D a bD c dD
(a+b D+c+d D) = = + = (a+b D)+(c+d D),
b+d a+c b a d c
and

     
ac + bdD (ad + bc)D a bD c dD
((a + b D) (c + d D)) = =
ad + bc ac + bdD b a d c

= (a + b D)(c + d D),

so is an homomorphism. Since K is a field, its ideals are {0} and K, so ker() is trivial or K.
Since (K) is clearly non-zero, then ker() 6= K and thus ker() = {0}. Hence, is injective.
Therefore, is an isomorphism of K with a subfield of M2 (Q).
Exercise 22. Let K1 and K2 be two finite extensions of a field F contained in the field K.
Prove that the F -algebra K1 F K2 is a field if and only if [K1 K2 : F ] = [K1 : F ][K2 : F ].
Solution: Define : K1 K2 K1 K2 by (a, b) = ab. We prove that is F -bilinear. Let
a, a1 , a2 K and b, b1 , b2 K2 . Then

((a1 , b) + (a2 , b)) = (a1 + a2 , b) = (a1 + a2 )b = a1 b + a2 b = (a1 , b) + (a2 , b),

and

((a,1 b) + (a, b2 )) = (a, b1 + b2 ) = a(b1 + b2 ) = ab1 + ab1 = (a, b1 ) + (a, b2 ).

We also have, for r F , (ar, b) = (ar)b = a(rb) = (rb). Therefore, is a F -bilinear map.
Hence, induces a F -algebra homomorphism : K1 F K2 K1 K2 . We use to prove both
directions. Note that K1 F K2 have dimension [K1 : F ][K2 : F ] as a vector space over F .
First, we suppose [K1 K2 : F ] = [K1 : F ][K2 : F ] and prove K1 F K2 is a field. In this
case K1 F K2 and K1 K2 have the same dimension over F . Let L = (K1 F K2 ). We claim
L = K1 K2 , i.e. is surjective. Note that L contains K1 and K2 . Since L is a subring of K1 K2
containing K1 (or K2 ), then L is a field (Exercise 16). Hence, L is a field containing both K1
and K2 . Since K1 K2 is the smallest such field (by definition), we have L = K1 K2 . Therefore
is surjective, as claimed. So, is an F -algebra surjective homomorphism between F -algebras
of the same dimension, hence is an isomorphism. Thus, K1 F K2 is a field.
Now suppose that K1 F K2 is a field. In this case is a field homomorphism. Therefore,
is either injective or trivial. It is clearly nontrivial since (1 1) = 1, so it is injective. Hence,
[K1 : F ][K2 : F ] [K1 K2 : F ]. As we already have [K1 K2 : F ] [K1 : F ][K2 : F ] (Proposition
21 of the book), the equality follows.

11
13.3 Classical Straightedge and Compass Constructions

13.3 Classical Straightedge and Compass Constructions


Exercise 1. Prove that it is impossible to construct the regular 9-gon.
Solution: Suppose the 9-gon is constructible. It has angles of 40 . Since we can bisect an
angle by straightedge and compass, the angle of 20 would be constructible. But then cos 20
and sin 20 would be constructible too, a contradiction (see proof of Theorem 24).
Exercise 2. Prove that Archimedes construction actually trisect the angle . [Note the
isosceles triangles in figure to prove that = = 2.]

Solution: Let O, P, Q and R be the points marked in the figure below.

Then = QP O, = RQO, = QRO, and is an exterior angle of 4P RO. Since


4P QO is isosceles, then = QP O = QOP . Since is an exterior angle of 4P QO, it
equals the sum of the two remote interior angles, i.e., equals QP O + QOP . This two angles
equals , hence = 2. Now, since 4QRO is isosceles, then = . Finally, since is an
exterior angle of 4P RO, equals the sum of the two remote interior angles, which are and .
Therefore, = + = + = 3.
Exercise 3. Prove that Conways construction indicated in the text actually constructs
2k 1/3 and 2k 2/3 . [One method: let (x, y) be the coordinates of the point C, a the distance
from
y 1k 2
B and C and b the distance from A to D; use similar triangles to prove (a) = , (b)
1 1+a
x b+k y 1 k2
= , (c) = , and also show that (d) (1 k 2 ) + (b + k)2 = (1 + a)2 ; solve
a 1+a xk 3k
these equations for a and b.]
Solution: We follow the hint. The distances a, b, x, y and x k are marked in the figure
below.

12
13.4 Splitting Fields and Algebraic Closures

From the figure, using similar triangles for (a), (b) and (c), and Pythagoras Theorem for
(d), the 4 relations are clear. Hence, we have

1 k2 b+k y 1 k2
y= , x=a , = and (1 k 2 ) + (b + k)2 = (1 + a)2 .
1+a 1+a xk 3k
3ky
So, 1 k 2 = y(1 + a) = xk implies 3k = (x k)(1 + a). From the equation for x above, we
find 3k = ( a(b+a)
1+a k)(1 + a) = a(b + k) k(1 + a), so b + k =
4k+ka
a . Using this in the last
equation and reducing, we get

(1 k 2 ) + (b + k)2 = (1 + a)2
4k + ka 2
(1 k 2 ) + ( ) = (1 + a)2
a
a2 (1 k 2 ) + (4k + ka)2 = a2 (1 + a)2
a2 (ka)2 + (4k)2 + 8k 2 a + (ka)2 = a2 + 2a3 + a4
a4 + 2a3 8k 2 a 16k 2 = 0.

We let a = 2y to obtain
h4 + h3 k 2 h k 2 = 0.
We find h = k 2/3 , hence a = 2k 2/3 . From b = 4k+ka
a k, we find b = 2k 1/3 . Therefore, we can
construct 2k 1/3 and 2k 2/3 using Conways construction.
Exercise 4. The construction of the regular 7-gon amounts to the constructibility of
cos(2/7). We have shall see later (Section 14.5 and Exercise 2 of Section 14.7) that =
2 cos(2/7) satisfies the equation x3 + x2 2x 1 = 0. Use this to prove that the regular 7-gon
is not constructible by straightedge and compass.
Solution: Let p(x) = x3 + x2 2x 1 and = 2 cos(2/7). By Rational Root Theorem,
if p has a root in Q, it must be 1 since it must divide its constant term. But p(1) = 1 and
p(1) = 1, so p is irreducible over Q. Therefore, is of degree 3 over Q, hence [Q() : Q]
cannot be a power of 2. Since we cant construct , the regular 7-gon is not constructible by
straightedge and compass.
Exercise 5. Use the fact that = 2 cos(2/5) satisfies the equation x2 + x 1 = 0 to
conclude that the regular 5-gon is constructible by straightedge and compass.
Solution: Let p(x) = x2 + x 1 = 0 and = 2 cos(2/5). By Rational Root Theorem,
if p has a root in Q, it must be 1. Since p(1) = 1 and p(1) = 1, p is irreducible over Q.
Hence, is of degree 2 over Q, so it is constructible. We can bisect an angle
pby straightedge
and compass, so = cos(2/5) is also constructible. Finally, as sin(2/5) = 1 cos2 (2/5),
sin(2/5) is also constructible. Therefore, the regular 5-gon is constructible by straightedge
and compass.

13.4 Splitting Fields and Algebraic Closures


Exercise 1. Determine the splitting field and its degree over Q for x4 2.

4 2. The roots of f are 4 2, 4 2, i 4 2 and i 4 2. Hence, the
Solution: Let f (x) = x
splitting
field
of f is Q(i, 4 2). So, the
splitting field of f has degree [Q(i, 4 2) : Q] = [Q(i, 4 2) :
Q( 4 2)][Q(

4
2) : Q] over Q. Since 4 2 is a root ofthe irreducible polynomial x4 2 overQ,
then [Q( 4 2) : Q] = 4. Furthermore, since i 6 Q( 4 2), then x2 + 1 is irreducible over Q( 4 2)

having i as a root, so [Q(i, 2) : Q( 2)] = 2. Therefore, [Q(i, 4 2) : Q] = 8.
4 4

Exercise 2. Determine the splitting field and its degree over Q for x4 + 2.

13
13.4 Splitting Fields and Algebraic Closures

Solution: Let f (x) = x4 + 2. Let K be the splitting field of f and let L be the splitting
field of x4 2, that is,L = Q(i,

4
2) (last exercise). We claim K = L, so that [K : Q] = 8 by
2 2
last exercise. Let = 2 + i 2 . First we prove L and K, then we prove K = L.
4 2 =

We prove
L. This is easy. Let = 2. Since L, then 2 L. We also have
i L, so 2, i L implies L.
We prove K. We have to prove i K and 2 K. Let be a root of x4 + 2, so that
4 = 2. Let be a root of x4 1, so that 4 = 1. Then ()4 = 4 4 = 2, hence is
also a root of x4 + 2. Since the roots of x4 1 are 1, i, the roots of x4 + 2 are and i.
Since K is generated over Q by there roots, then i/ = i K. Now let = 2 K. Since
2 = 4 = 2, then is a root of x2 + 2. Since the roots of 2
x + 2 are i 2 and i 2, then
is one of this roots. In either case /i K, which implies 2 K. Therefore, K.
Now we prove L = K proving both inclusions.
Let be a root of x4 + 2 and be a root of x4 2. Then 4 = 2 and 4 = 2. Note that
2 = i, so 4 = 1. Hence, ()4 = 4 4 = 2, so is a root of x4 + 2. Then, as we proved
earlier, the roots of x4 + 2 are and i. We also have ()4 = 4 4 = 2, so is a root
of x4 2. Then, by last exercise, the roots of x4 2 are and i. Now, since and
are in K, we have K. We also have i K, so all roots of x4 2 are in K. Since L is
generated by these roots, then L K. Similarly, and are in L, so L; since i L, then
all roots of x4 + 2 are in L. Since K is generated by these roots, we have K L. Therefore,
K = L, so [K : Q] = 8.
Exercise 3. Determine the splitting field and its degree over Q for x4 + x2 + 1.
Solution:
Let f (x) = x4 +x2 + 1. Note that f (x) = (x2 + x + 1)(x2 x + 1), so the roots of
f are 2 i 2 . Let w = 12 i 23 , so this roots are w, w, w, w, where w denotes the complex
1 3

conjugate of w (i.e., w = 12 + i 23 ). Hence, the splitting field of f is Q(w, w). Since w + w = 1,
then Q(w, w) = Q(w). Furthermore, w is a root of x2 x + 1, that is irreducible over Q since
w 6 Q. Therefore, the degree of the splitting field of f is [Q(w) : Q] = 2.
Exercise 4. Determine the splitting field and its degree over Q for x6 4.
6 4. Note that f (x) = (x3 2)(x3 + 2). The roots of x3 2 are 3 2,

Solution:
Let f (x) = x
3 2 and 2 3 2, where is the primitive 3rd root of unity, i.e., = exp(2i/3) = cos(2/3) +
1

3

i sin(2/3) = 2 + 2 . Furthermore, the roots of x3 +2 are 3 2, 3 2 and 2 3 2. Therefore,

the splitting
field of f is Q(, 3 2). Then [Q(, 3 2) : Q] = [Q(, 3 2) : Q(
3
2)][Q( 3 2) : Q]. We
have that 3 2 is a root of the irreducible polynomial x3 2 over Q, so 3 2 has degree 3 over Q.
Furthermore, is a root of x2 + x + 1, irreducible over Q( 3
2), so has degree 2 over Q( 3
2).
Hence, the degree of the splitting field of f is [Q(, 3 2) : Q] = 6.
Exercise 5. Let K be a finite extension of F . Prove that K is a splitting field over F if
and only if every irreducible polynomial in F [x] that has a root in K splits completely in K[x].
[Use Theorems 8 and 27.]
Solution: We follow the hint. First suppose that K is a splitting field over F . Hence, there
exists f (x) F [x] such that K is the splitting field of f . Let g(x) be an irreducible polynomial
in F [x] with a root K. Let be any root of g. We prove K, so that g splits completely

in K[x]. By Theorem 8, there is an isomorphism : F () F () such that () = .
Furthermore, K() is the splitting field for f over F (a), and K() is the splitting field for f

over F (). Therefore, by Theorem 28, extends to an isomorphism : K() K(). Since
K = K(), then [K : F ] = [K() : F ] = [K() : F ], so K = K(). Thus, K.
Now suppose that every irreducible polynomial in F [x] that has a root in K splits completely
in K[x]. Since [K : F ] is finite, then K = F (1 , . . . , n ) for some 1 , . . . , n . For every
1 i n, let pi be the minimal polynomial of i over F , and let f = p1 p2 pn . Since every

14
13.5 Separable and Inseparable Extension

i is in K, every pi has a root in K, hence splits completely in K. Therefore, f splits completely


in K and K is generated over F by its roots, so K is the splitting field of f (x) F [x].
Exercise 6. Let K1 and K2 be finite extensions of F contained in the field K, and assume
both are splitting fields over F .
(a) Prove that their composite K1 K2 is a splitting field over F .
(b) Prove that K1 K2 is a splitting field over F . [Use the preceding exercise.]
Solution: (a) Let K1 be the splitting field of f1 (x) F [x] over F and K2 the splitting field
of f2 (x) F [x] over F . Thus, K1 is generated over F by the roots of f1 , and K2 is generated
over F by the roots of f2 . Therefore, f1 f2 splits completely in K1 K2 and K1 K2 is generated
over F by its roots, hence is the splitting field of f1 f2 (x) F [x].
(b) We follow the hint. By last exercise, we have to prove that every irreducible polynomial
in F [x] that has a root in K1 K2 splits completely in (K1 K2 )[x]. So, let f (x) be an
irreducible polynomial in F [x] that has a root, say , in K1 K2 . By last exercise, f splits
completely in K1 and splits completely in K2 . Since K1 and K2 are contained in K, by the
uniqueness of the factorization of f in K, the roots of f in K1 must coincide with its roots in
K2 . Hence, f splits completely in (K1 K2 )[x].

13.5 Separable and Inseparable Extension


Exercise 1. Prove that the derivative Dx of a polynomial satisfies Dx (f (x) + g(x)) =
Dx (f (x))+Dx (g(x)) and Dx (f (x)g(x)) = Dx (f (x))g(x)+Dx (g(x))f (x) for any two polynomials
f (x) and g(x).
Solution: Let f (x) = an xn + + a1 x + a0 and g(x) = bm xm + + b1 x + b0 be two
polynomials. Suppose, without any loss of generality, that n m. Thus, we can write g(x) =
bn xn + + b1 x + b0 , where some of the last coefficients bi could be zero. We have f (x) + g(x) =
(an + bn )xn + + (a1 + b1 )x + (a0 + b0 ), so

Dx (f (x) + g(x)) = n(an + bn )xn1 + + 2(a2 + b2 )x + (a1 + b1 ) = Dx (f (x)) + Dx (g(x)).

Now we prove the formula for the product. Let cn = nk=0 ak bnk , so that
P

Xn Xn 2n X
X l 2n
X
f (x)g(x) = ( ak xk )( bk xk ) = ( ak blk )xl = cl xl .
k=0 k=0 l=0 k=0 l=0

Hence,
2n
X 2n1
X
l
Dx (f (x)g(x)) = Dx ( cl x ) = (l + 1)cl+1 xl ,
l=0 l=0

so the coefficient of xl in Dx (f (x)g(x)) is (l + 1)cl+1 .


Now, we have Dx (f (x)) = nan xn1 + +2a2 x+a1 , and Dx (g(x)) = nbn xn1 + +2b2 x+b1 .
So (recall product of polynomials, page 295 of the book)
n
X Xn
Dx (f (x))g(x) = ( kak xk1 )( bk xk )
k=1 k=0
n1
X Xn 2n1
X X l
=( (k + 1)ak+1 xk )( bk xk ) = ( (k + 1)ak+1 blk )xl ,
k=0 k=0 l=0 k=0

15
13.5 Separable and Inseparable Extension

and
Xn Xn
f (x)Dx (g(x)) = ( ak xk )( kbk xk1 )
k=0 k=1
Xn n1
X 2n1
X l
X
k k
=( ak x )( (k + 1)bk+1 x ) = ( ak (l k + 1)blk+1 )xl .
k=0 k=0 l=0 k=0

Therefore, the coefficient of xl in Dx (f (x))g(x) + Dx (g(x))f (x) is


l
X l
X
( (k + 1)ak+1 blk ) + ( (l k + 1)ak blk+1 )
k=0 k=0
l1
X l
X
= (l + 1)al+1 b0 + ( (k + 1)ak+1 blk ) + ( (l k + 1)ak blk+1 ) + (l + 1)a0 bl+1
k=0 k=1
Xl l
X
= (l + 1)al+1 b0 + ( kak blk+1 ) + ( (l k + 1)ak blk+1 ) + (l + 1)a0 bl+1
k=1 k=1
Xl
= (l + 1)al+1 b0 + ( (l + 1)ak blk+1 ) + (l + 1)a0 bl+1
k=1
l+1
X
= (l + 1)( ak blk+1 ) = (l + 1)cl+1 .
k=0

Since all their coefficients are equal, we conclude Dx (f (x)g(x)) = Dx (f (x))g(x)+Dx (g(x))f (x).
Exercise 2. Find all irreducible polynomials of degree 1, 2 and 4 over F2 and prove that
their product is x16 x.
Solution: The polynomials x and x + 1 are the only (non-constant, i.e. 6= 0, 1) polynomials
of degree 1 over F2 ; they are clearly irreducible. A polynomial f (x) F2 [x] of degree 2 is
irreducible over F2 if and only if it does not have a root in F2 , that is, exactly when f (0) =
f (1) = 1. Hence, the only irreducible polynomial of degree 2 over F2 is x2 + x + 1. Now, for
a polynomial f (x) F2 [x] of degree 4 to be irreducible, it must have no linear or quadratic
factors. We can also apply the condition f (1) = f (0) = 1 to discard the ones with linear factors.
Furthermore, f must have an odd number of terms (or it will be 0), and must have constant
term 1 (or x will be a factor). We are left with

x4 + x3 + x2 + x + 1 x4 + x3 + 1
x4 + x2 + 1 x4 + x + 1.

For any of this polynomials to be irreducible, it cant be factorized as two quadratic irreducible
factors. Since x2 +x+1 is the only irreducible polynomial of degree 2 over F2 , only (x2 +x+1)2 =
x4 + x2 + 1 of this four is not irreducible. Hence, the irreducible polynomials of degree 4 over
F2 are x4 + x3 + x2 + x + 1, x4 + x3 + 1 and x4 + x + 1.
Now, since x + 1 = x 1 in F2 , we have (x + 1)(x4 + x3 + x2 + x + 1) = x5 1. We also
calculate (x2 + x + 1)(x4 + x + 1)(x4 + x3 + 1) = x10 + x5 + 1. So, the product of all this
irreducible polynomials is

x(x + 1)(x2 + x + 1)(x4 + x + 1)(x4 + x3 + 1)(x4 + x3 + x2 + x + 1)


= x(x5 1)(x10 + x5 + 1) = x16 x.

16
13.5 Separable and Inseparable Extension

Exercise 3. Prove that d divides n if and only if xd 1 divides xn 1. [Note that if


n = qd + r then xn 1 = (xqd+r xr ) + (xr 1).]
Solution: We follow the hint. Suppose d divides n, so that n = qd for some q Z. Then
xn 1 = xqd 1 = (xd 1)(xqdd + xqd2d + . . . + xd + 1). So xd 1 divides xn 1.
Conversely, suppose d does not divide n. Then n = qd + r for some q, r Z with 0 < r < d.
Thus xn 1 = (xqd+r xr ) + (xr 1) = xr (xqd 1) + (xr 1) = xr (xd 1)(xqdd + xqd2d +
. . . + xd + 1) + (xr 1). Since xd 1 divides the first term, but doesnt divide xr 1 (as r < d),
then xd 1 does not divide xn 1.
Exercise 4. Let a > 1 be an integer. Prove for any positive integers n, d that d divides
n if and only if ad 1 divides an 1 (cf. the previous exercise). Conclude in particular that
Fpd Fpn if and only if d divides n.
Solution: The first assertion is analogous to the last exercise.
d
Now, Fpd is defined as the field whose pd elements are the roots of xp x over Fp . Similarly
is defined Fpn . Take a = p. So, d divides n if and only if pd 1 divides pn 1, and that occurs
d n
exactly when xp 1 1 divides xp 1 1 (by last exercise). Thus, if d divides n, any root of
d d n n
xp x = x(xp 1 1) must be a root of xp x = x(xp 1 1), hence Fpd Fpn . Conversely,
d n
if Fpd Fpn , then xp 1 1 divides xp 1 1, so d divides n.
Exercise 5. For any prime p and any nonzero a Fp prove that xp x + a is irreducible
and separable over Fp . [For the irreducibility: One approach prove first that if is a root
then + 1 is also a root. Another approach suppose its reducible and compute derivatives.]
Solution: Let f (x) = xp x + a. Let be a root of f (x). First we prove f is separable.
Since ( + 1)p ( + 1) + a = p + 1 1 + a = 0, then + 1 is also a root of f (x). This
gives p distinct roots of f (x) given by + k with k Fp , so f is separable.
Now we prove f is irreducible. Let f = f1 f2 fn where fi (x) Fp [x] is irreducible for all
1 i n. Let 1 i < j n and let i be a root of fi and j be a root of fj , so that fi is the
minimal polynomial of i and fj the minimal polynomial of j . We prove deg fi = deg fj . Since
i is a root of fi , it is a root of f , hence there exists k1 Fp such that i = + k1 . Similarly,
there exists k2 Fp such that j = +k2 . Thus, i = j +k1 k2 , so fi (x+k1 k2 ) is irreducible
having j as a root, so it must be its minimal polynomial. Hence, fi (x + k1 k2 ) = fj (x), so
deg fi = deg fj , as claimed. Since i and j were arbitrary, then all fi are of the same degree, say
q. Then p = deg f = nq, so n = 1 or n = p (as p is prime). If n = p, then all roots of f are in
Fp , so Fp and thus 0 = p + a = a, contrary to the hypothesis. Therefore, n = 1, so f
is irreducible.
n n
Exercise 6. Prove that xp 1 1 = Fn (x ). Conclude that Fn = (1)p so
Q Q
p p
the product of the nonzero elements of a finite field is +1 if p = 2 and 1 is p is odd. For p
odd and n = 1 derive Wilsons Theorem: (p 1)! 1 (mod p).
n
Solution: By definition, Fpn is the field whose pn elements are the roots of xp x over
n n
Fp . Since xp 1 = x(xp 1 1), clearly
n
Y
xp 1 1 = (x ).
F
pn

17
13.5 Separable and Inseparable Extension

Set x = 0. Then Y n 1
Y
1 = () = (1)p
F
pn F
pn
n 1 n 1 n 1
Y
(1)p (1) = (1)p (1)p
F
pn
n
Y
(1)p = .
F
pn

Hence, the product of the nonzero elements is +1 if p = 2 and 1 is p is odd. For p odd and
n = 1, we have Y
1 = ,
F
p

so taking module p we find [1][2] [p 1] = [1], i.e., (p 1)! 1 (mod p).


Exercise 7. Suppose K is a field of characteristic p which is not a perfect field: K 6=
K p . Prove there exist irreducible inseparable polynomials over K. Conclude that there exist
inseparable finite extensions of K.
Solution: Let a K such that a 6= bp for every b K. Let f (x) = xp a. We prove that f
is irreducible and inseparable. If is a root of xp a, then xp a = (x )p , so is a multiple
root of f (with multiplicity p), hence f is inseparable. Now, let g(x) be an irreducible factor of
f (x). Note that 6 K, otherwise a = p , contrary to the hypothesis. Then g(x) = (x )k
for some k p. Using the binomial theorem, we have

g(x) = (x )k = xk kxk1 + + ()k .

Therefore, k K. Since 6 K, then k = p, so g = f . Hence, f is irreducible. We conclude


that K() is an inseparable finite extension of K.
Exercise 8. Prove that f (x)p = f (xp ) for any polynomial f (x) Fp [x].
Solution: Let f (x) = an xn + . . . + a1 x + a0 Fp [x]. Since Fp has characteristic p, then
(a + b)p = ap + bp for any a, b Fp . Easily we can generalize this to a finite number of terms, so
that (x1 + + xn )p = xp1 + + xpn for any x1 , , xn Fp . Furthermore, by Fermats Little
Theorem, ap = a for every a Fp . So, over Fp , we have

f (x)p = (an xn + . . . + a1 x + a0 )p = apn xnp + . . . + ap1 xp + ap0 = an xnp + . . . + a1 xp + a0 = f (xp ).

Exercise 9. Show that the binomial coefficient pn pi



pi is the coefficient of x in the expansion
of (1 + x)pn . Working p i p n
 over Fp show that this is the coefficient of (x ) in (1 + x ) and hence
pn n

prove that pi i (mod p).
Solution: By the binomial theorem, we have
pn  
pn
X pn i
(1 + x) = x,
i
i=0

so the coefficient of xpi in the expansion of (1 + x)pn is pn



pi .
Since Fp has characteristic p, we have (1 + x)pn = 1 + xpn = (1 + xp )n , so over Fp this is the
coefficient of (xp )i in (1 + xp )n . Furthermore, (1 + x)pn = (1 + xp )n implies
n   pn  
p n
X n X
p i pn i
(1 + x ) = (x ) = x = (1 + x)pn
i k
i=0 i=0

18
13.6 Cyclotomic Polynomials and Extensions

pn n
 
over Fp , hence pi i (mod p).
Exercise 10. Let f (x1 , x2 , . . . , xn ) Z[x1 , x2 , . . . , xn ] be a polynomial in the variables
x1 , x2 , . . . , xn with integer coefficients. For any prime p prove that the polynomial

f (x1 , x2 , . . . , xn )p f (xp1 , xp2 , . . . , xpn ) Z[x1 , x2 , . . . , xn ]

has all its coefficients divisible by p.


Solution: This is equivalent to prove that for any prime number p, we have f (x1 , x2 , . . . , xn )p =
f (xp1 , xp2 , . . . , xpn ) in Fp [x1 , x2 , . . . , xn ]. Let

a1 ,...,n x11 . . . xnn


X
f (x1 , x2 , . . . , xn ) =
1 ,...,n =0

be an element of Fp [x1 , x2 , . . . , xn ].
Since Fp has characteristic p, then (x1 + + xn )p = xp1 + + xpn for any x1 , , xn Fp .
Furthermore, by Fermats Little Theorem, ap = a for every a Fp . Hence, over Fp we have

a1 ,...,n x11 . . . xnn )p = (a1 ,...,n x11 . . . xnn )p


X X
f (x1 , x2 , . . . , xn )p = (
ap1 ,...,n (x11 . . . xnn )p = a1 ,...,n (xp p p
X X
1 pn p
= 1 . . . xn ) = f (x1 , x2 , . . . , xn ).

Exercise 11. Suppose K[x] is a polynomial ring over the field K and F is a subfield of K.
If F is a perfect field and f (X) F [x] has no repeated irreducible factors in F [x], prove that
f (x) has no repeated irreducible factors in K[x].
Solution: Let f (x) F [x] with no repeated irreducible factors in F [x]. We can suppose f
is monic. Then f = f1 f2 fn for some monic irreducible polynomials fi (x) F [x]. Since F
is perfect, f is separable, hence all fi has distinct roots. Thus, f splits in linear factors in the
closure of F , hence splits in linear factors in the closure of K. Therefore, f (x) has no repeated
irreducible factors in K[x].

13.6 Cyclotomic Polynomials and Extensions


Exercise 1. Suppose m and n are relatively prime positive integers. Let m be a primitive
mth root of unity and let n be a primitive nth root of unity. Prove that m n is a primitive
mnth root of unity.
Solution: Since (m n )mn = 1, then m n is an mnth root of unity. Now, let 1 k < mn.
Then (m n )k = mk k . For a contradiction, suppose k k = 1. Then, k = 1 and k = 1,
n m n m n
hence m divides k and n divides k, so mn divides k (as m 6= n). Since 1 k < mn, this is
impossible. So, (m n )k 6= 1 for all 1 k < mn and thus the order of m n is mn. Therefore,
m n generates the cyclic group of all mnth roots of unity, that is, m n is a primitive mnth
root of unity.
Exercise 2. Let n be a primitive nth root of unity and let d be a divisor of n. Prove that
nd is a primitive (n/d)th root of unity.
Solution:
Since (nd )(n/d) = nn = 1, then nd is an (n/d)th root of unity. Now let 1 k < (n/d). Then
(n )k = nkd . Since 1 kd < n, then nkd 6= 1, so (nd )k 6= 1. Hence, the order of nd is (n/d), so
d

it generates the cyclic group of all (n/d)th roots of unity, that is, nd is a primitive (n/d)th root
of unity.
Exercise 3. Prove that if a field contains the nth roots of unity for n odd then it also
contains the 2nth roots of unity.

19
13.6 Cyclotomic Polynomials and Extensions

Solution: Let F be a field that contains the nth roots of unity for n odd and let be a
2nth root of unity. If n = 1, then F , so suppose n 6= 1. Since 2n = 1, then n is a root
of x 1. Since the roots of this polynomial are 1 and 1, and n 6= 1, then n = 1. Hence,
2

()n = (1)n ()n = (1)n+1 = 1 (since n is odd), so F . Since F is a field, then F .


Exercise 4. Prove that is n = pk m where p is a prime and m is relatively prime to p then
there are precisely m distinct nth roots of unity over a field of characteristic p.
Solution: Let F be a field with char F = p. The roots of unity over F are the roots of
k k
xn 1 = xp m 1 = (xm 1)p , so are the roots of xm 1. Now, since m is relatively prime
to p, so is xm 1 and its derivative mxm1 , so xm 1 has no multiple roots. Hence, the m
different roots of xm 1 are precisely the m distinct nth roots of unity over F .
Exercise 5. Prove there are only a finite number of roots of unity in any finite extension
K of Q.

Solution: We use the inequality (n) n/2 for all n 1. Let K be an extension of Q
with infinitely many roots of unity. Let N N. Then, there exits n N such that n > 4N 2

and there exits some nth root of unity K. Then [K : Q] [Q() : Q] = (n) n/2 > N .
Since N was arbitrary, we conclude that [K : Q] > N for all N N, so [K : Q] is infinite.
Therefore, in any finite extension of Q there are only a finite number of roots of unity.
Exercise 6. Prove that for n odd, n > 1, 2n (x) = n (x).
Solution: Since 2n (x) and n (x) are irreducible, they are the minimal polynomial of
any of its roots. Thus, it is sufficient to find a common root for both. Let 2 be the primitive 2th
root of unity and let n be a primitive nth root of unity. Note that 2 = 1, so that 2 n = n .
Since n is odd, 2 and n are relatively prime. So, by Exercise 1, 2 n is a primitive 2nth root
of unity, i.e, a root of 2n (x). Furthermore, n is clearly a root of n (x). Thus, n is a
common root for both 2n (x) and n (x), hence 2n (x) = n (x).
Exercise 7. Use the Mbius Inversion Formula indicated in Section 14.3 to prove
Y
n (x) = (xd 1)(n/d) .
d|n

Solution: The Mbius Inversion FormulaPsates that if f (n) is defined for all nonnegative
integers and F (n) = d|n f (d), then f (n) = d|n (d)F ( nd ). So lets start with the formula
P

Y
xn 1 = d (x).
d|n

We take natural logarithm in both sides and obtain


Y X
ln(xn 1) = ln( d (x)) = ln d (x).
d|n d|n

So, we use the Mbius Inversion Formula for f (n) = ln n (x) and F (n) = ln(xn 1) to obtain
X X
ln n (x) = (d) ln(xn/d 1) = ln(xn/d 1)(d) .
d|n d|n

Hence, taking exponentials we obtain


X Y Y
n (x) = exp( ln(xn/d 1)(d) ) = (xn/d 1)(d) = (xd 1)(n/d) .
d|n d|n d|n

20
13.6 Cyclotomic Polynomials and Extensions

`1
Exercise 8. Let ` be a prime and let ` (x) = xx1 = x`1 + x`2 + + x + 1 Z[x] be the
th
` cyclotomic polynomial, which is irreducible over Z by Theorem 41. This exercise determines
the factorization of ` (x) modulo p for any prime p. Let denote any fixed primitive `th root
of unity.
(a) Show that if p = ` then ` (x) = (x 1)`1 F` [x]
(b) Suppose p 6= ` and let f denote the order of p mod ` , i.e., f is the smallest power of p
with pf 1 mod `. Use the fact that F pn is a cyclic group to show that n = f is the smallest
n
power p of p with Fp . Conclude that the minimal polynomial of over Fp has degree f .
n

(c) Show that Fp () = Fp ( a ) for any integer a not divisible by `. [One inclusion is obvious.
For the other, note that = ( a )b where b is the multiplicative inverse of a mod `.] Conclude
using (b) that, in Fp [x], ` (x) is the product of `1
f distinct irreducible polynomials of degree
f.
(d) In particular, prove that, viewed in Fp [x], 7 (x) = x6 + x5 + + x + 1 is (x 1)6 for
p = 7, a product of distinct linear factor for p 1 mod 7, a product of 3 irreducible quadratics
for p 6 mod 7, a product of 2 irreducible cubics for p 2, 4 mod 7, and is irreducible for
p 3, 5 mod 7.
Solution: `
`1
(a) Since p is prime, in Fp [x] we have (x1)p = xp 1, so ` (x) = xx1 = (x1) x1 = (x1)
l1 .

(b) Note that has order ` as being a primitive `th root of unity. Since pf 1 mod `,
f
then pf 1 = q` for some integer q, hence p 1 = q` = 1, so Fpf . Now we prove that
f is the smallest integer with that property. Suppose Fpn for some n. Then is a root of
n
xp 1 1, hence ` divides pn 1 (see Exercise 13.5.3). Since f is the smallest power of p such
that pf 1 mod `, is the smallest integer such that ` divides pf 1, so n l, as desired. This
in fact proves that Fp () = Fpf , so the minimal polynomial of over Fp has degree f .
(c) Since a Fp (), clearly Fp ( a ) Fp (). For the other direction we follow the hint. Let
b the the multiplicative inverse of a mod `, i.e ab 1 mod `. Then ( a )b = , so Fp ( a )
and thus Fp () Fp ( a ). The equality follows.
Now, consider ` (x) as a polynomial over Fp [x]. Let i for 1 i ` be ` distinct primitive
`th roots of unity. The minimal polynomial of each i has degree f by part (b). Hence, the
irreducible factors of ` (x) have degree f . Since ` have degree ` 1, then there must be `1 f
factors, and all of them are different since ` (x) is separable.
(d) If p = 7, then 7 (x) = (x 1)6 by part (a). If p 1 mod 7, then f = 1 in (b) and
all roots have degree 1, so 7 (x) splits in distinct linear factors. If p 6 mod 7, then f = 2 is
the smallest integer such that pf = p2 36 1 mod 7, so we have 3 irreducible quadratics. If
p 2, 4 mod 7, then f = 3 is the smallest integer such that p3 23 , 43 8, 64 1 mod 7, so
we have 2 irreducible cubics. Finally, if p 3, 5 mod 7, then f = 6 is the smallest integer such
that p6 36 , 56 729, 15626 1 mod 7, hence we have an irreducible factor of degree 6.
Exercise 9. Suppose A is an n by n matrix over C for which Ak= I  for some integer
1
k 1. Show that A can be diagonalized. Show that the matrix A = where is an
0 1
element of a field of characteristic p satisfies Ap = I and cannot be diagonalized if 6= 0.
Solution: Let A be an n by n matrix over C for which Ak = I for some integer k 1.
Then the minimal polynomial of A divides xk 1. Since we are working over C, there are k
distinct roots of this polynomial, so the minimal polynomial of A can be split in linear factors.
Hence, A is diagonalizable.
 
1
Now consider A = where is an element of a field of characteristic p.
0 1
 
n 1 n
Computing powers of A, we can prove (by induction) that A = for every positive
0 1

21
13.6 Cyclotomic Polynomials and Extensions

integer n. Since p = 0, then Ap = I. Now, if A is diagonalizable, there exists some non-singular


matrix P such that A = P DP 1 , where D is a diagonal matrix whose diagonal entries are the
eigenvalues of A. Since A has only one eigenvalue 1, then D = I, and thus A = P IP 1 = I.
So, if A is diagonalizable, it must be = 0.
Exercise 10. Let denote the Frobenius map x 7 xp on the finite field Fpn . Prove that
gives an isomorphism of Fpn to itself (such an isomorphism is called an automorphism). Prove
that n is the identity map an that no lower power of is the identity.
Solution: Let a, b Fpn . Then (a + b) = (a + b)p = ap + bp = (a) + (b), and
(ab) = (ab)p = ap bp = (a)(b), so is and homomorphism. Moreover, if (a) = 0, then
ap = 0 implies a = 0. Hence, is injective. Since Fpn is finite, then is also surjective
n
so it is an isomorphism. Furthermore, since every element of Fpn is a root of xp x, then
n
n (a) = ap = a for all a Fpn , so n is the identity map. Now, let m be an integer such that
m
m is the identity map. Then ap = a for all a Fpn , so every element of Fpn must be a root
m
of xp x. Hence, Fpn Fpm and thus n divides m (Exercise 13.5.4), so n m.
Exercise 11. Let denote the Frobenius map x 7 xp on the finite field Fpn as in the
previous exercise. Determine the rational canonical form over Fp for considered as an Fp -linear
transformation of the n-dimensional Fp -vector space Fpn .
Solution: Note that the minimal polynomial of is xn 1, for if satisfies some polynomial
xn1 + + a pn1 + + a xp + a
1 x + a0 of degree n 1 (or less) with coefficients in Fp , then x 1 0
for all x Fpn , which is impossible. Since Fpn has degree n as a vector space over Fp , then
xn 1 is also the characteristic polynomial of , hence is the only invariant factor. Therefore,
the rational canonical form of over Fp is the companion matrix of xn 1, which is

0 0 0 1
1 0 0 0

0 1 0 0
.
.. .. . . .. ..
. . . . .
0 0 1 0

Exercise 12. Let denote the Frobenius map x 7 xp on the field Fpn as in the previous
exercise. Determine the Jordan canonical form (over a field containing all the eigenvalues) for
considered as an Fp -linear transformation of the n-dimensional Fp -vector space Fpn .
Solution: Well work over the algebraic closure of Fpn , to ensure the field to contain all
eigenvalues. In last exercise we proved that the minimal and characteristic polynomial of is
xn 1. Moreover, the eigenvalues of are the nth roots of unity. We use Exercise 4 and write
k
n = pk m for some prime p and some m relatively prime to p, so that xn 1 = (xm 1)p
and we get exactly m distinct nth roots of unity, each one with multiplicity pk . Since all the
eigenvalues are zeros of both the minimal and characteristic polynomial of multiplicity pk , we
get m Jordan blocks of size pk . Now, fix a primitive mth root of unity, say . Then, each Jordan
block each of the form i
1 0 0 0
0 i 1 0 0

0 0 i 0 0
Ji = . . . .

. . . . .. ..
. . .
. . .
0 0 0 i 1
0 0 0 0 i

22
13.6 Cyclotomic Polynomials and Extensions

for some 0 i m 1. We already know the Jordan canonical form is given by



J0 0 0
0 J1 0

0 0 0
.
.. .. . . ..
. . . .
0 0 Jm1

Exercise 13. (Wedderburns Theorem on Finite Division Rings) This exercise outlines a
proof (following Witt) of Wedderburns Theorem that a finite division ring D is a field (i.e., is
commutative).
(a) Let Z denote the center of D (i.e., the elements of D which commute with every element
of D). Prove that Z is a field containing Fp for some prime p. If Z = Fq prove that D has order
q n for some integer n [D is a vector space over Z].
(b) The nonzero elements D of D form a multiplicative group. For any x D show that
the elements of D which commute with x form a division ring which contains Z. Show that
this division ring if of order q m for some integer m and that m < n if x is not an element of Z.
(c) Show that the class equation (Theorem 4.7) for the group D is
r
n
X qn 1
q 1 = (q 1) +
|CD (xi )|
i=1

where x1 , x2 , . . . , xr are representatives of the distinct conjugacy classes in D not contained


in the center of D . Conclude from (b) that for each i, |CD (xi )| = q mi 1 for some m1 < n.
qn 1
(d) Prove that since m is an integer (namely, the index |D : CD (xi )|) then mi and
q i 1
divides n (cf. Exercise 4 of Section 5). Conclude that n (x) divides (xn 1)/(xmi 1) and
hence that the integer n (q) divides (q n 1)/(q mi Q 1) for i = 1, 2, . . . , r.
(e) Prove that (c) and (d) imply that n (q) = primitive (q ) divides q 1. Prove that
|q | > q 1 (complex absolute value) for any root of unity 6= 1 [note that 1 is the closest
point on the unit circle in C to the point q on the real line]. Conclude that n = 1, i.e., that
D = Z is a field.
Solution: (a) Z is a division subring of D, and it is commutative by definition of the center,
so Z is a field. Since it is finite, its prime subfield is Fp for some prime p, so it is isomorphic to
Fpm for some integer m. Let q = pn , so that Z is isomorphic to Fq . Since D is a vector space
over Z, then |D| = q n for some integer n.
(b) Let x D and let CD (x) be the set of the elements in D that commutes with x.
Clearly Z CD (x). We prove that every element a CD (x) has an inverse in CD (x). Since
a CD (x), then ax = xa. Since D is a division ring, then a1 D. Moreover, we have
a1 ax = a1 xa and thus x = a1 xa, so xa1 = a1 x and a1 CD (x). Hence, CD (x) is a
division ring. Now, since Z CD (x), then CD (x) is a Z-vector space, so |CD (x)| = q m for
some integer m. If x 6 Z, then CD (x) is a proper subset of D and hence m < n.
(c) The class equation for the group D is
r
X
|D | = |Z(D )| + |D : CD (xi )|,
i=1

where the xi are the representatives of the distinct conjugacy class. By (a) we have |D | = q n 1,
qn 1 qn 1
|Z(D )| = q 1 and |CD (xi )| = q mi 1. Then |D : CD (xi )| = = m .
|CD (xi )| q i 1

23
13.6 Cyclotomic Polynomials and Extensions

Replacing in the class equation we obtain


r r
X qn 1 X qn 1
q n 1 = (q 1) + = (q 1) + .
|CD (xi )| q mi 1
i=1 i=1

qn 1
(d) Since |D : CD (xi )| is an integer, then |D : CD (xi )| = is an integer. Hence,
q mi 1
q mi 1 divides q n 1, so (Exercise 13.5.4) mi divides n. Since mi < n (no xi is in Z), no
mth th root of unity. Therefore, as (x) divides xn 1, it must divide
i root of unity is a n n
n m
(x 1)/(x 1) for i = 1, 2, . . . , r.. Letting x = q we have n (q) divides (q n 1)/(q mi 1)
i

for i = 1, 2, . . . , r.
(e) From (d), n (q) divides (q n 1)/(q mi 1) for i = 1, 2, . . . , r, so the class equation in
(c) implies n (q) divides q 1. Now, let 6= 1 be a nth root of unity.
Q In the complex plane q is
closer to 1 that is, so |q | > |q 1| = q 1. Since n (q) = primitive (q ) divides q 1,
this is impossible unless n = 1. Hence, D = Z and D is a field.
Exercise 14. Given any monic polynomial P (x) Z[x] of degree at leat one show that
there are infinitely many distinct prime divisors of the integers

P (1), P (2), P (3), . . . , P (n), . . .

[Suppose p1 , p2 , . . . , pk are the only primes dividing the values P (n), n = 1, 2, . . .. Let N be
an integer with P (N ) = a 6= 0. Show that Q(x) = a1 P (N + ap1 p2 . . . pk x) is an element
of Z[x] and that Q(n) 1 (mod p1 p2 . . . pk ) for n = 1, 2, . . .. Conclude that there is some
integer M such that Q(M ) has a prime factor different from p1 , p2 , . . . , pk and hence that
P (N + ap1 p2 . . . pk M ) has a prime factor different from p1 , p2 , . . . pk .]
Solution: We follow the hint. Let P (x) = xn + + a1 x + a0 be a monic polynomial of
degree 1 over Z. For a contradiction, suppose there are only finitely many primes dividing
the values P (n), n = 1, 2, . . ., say p1 , p2 , . . . , pk . Let N be an integer such that P (N ) = a 6= 0.
Let Q(x) = a1 P (N + ap1 p2 . . . pk x). Then, using the binomial theorem, we have

Q(x) = a1 P (N + ap1 p2 . . . pk x)
= a1 ((N + ap1 p2 . . . pk x)n + + a1 (N + ap1 p2 . . . pk x) + a0 )
= a1 (N n + an1 N n1 + + a1 N + a0 + R(x))
= a1 (P (N ) + R(x))
= 1 + a1 R(x)

for some polynomial R(x) Z[x] divisible by ap1 p2 . . . pk . Hence, Q(x) Z[x]. Moreover, for
all n Z+ , P (N +ap1 p2 . . . pk n) a (mod p1 , p2 , . . . , pk ), so Q(n) = a1 P (N +ap1 p2 . . . pk n)
a1 a = 1 (mod p1 , p2 , . . . , pk ). Now let m be a positive integer such that |Q(m)| > 1, so that
Q(m) 1 (mod pi ) for all i. Therefore, none of the pi s divide Q(m). Since |Q(m)| > 1,
there exists a prime q 6= pi for all i such that q divides Q(m). Then q divides aQ(m) =
P (N + ap1 p2 . . . pk m), contradicting the fact that only the primes p1 , p2 , . . . , pk divides the
numbers P (1), P (2), . . ..
Exercise 15. Let p be an odd prime not dividing m and let m (x) be the mth cyclotomic
polynomial. Suppose a Z satisfies m (a) 0 (mod p). Prove that a is relatively prime to p
and that the order of a in (Z/pZ) is precisely m. [Since
Y Y
xm 1 = d (x) = m (x) d (x)
d|n d|n
d<n

24
13.6 Cyclotomic Polynomials and Extensions

we see first that am 1 0 (mod p) i.e., a 1 (mod p). If the order of a mod p were less that
m, then ad 1 (mod p) for some d dividing m, so then d (a) 0 (mod p) for some d < m.
but then xm 1 would have a as a multiple root mod p, a contradiction.]
Solution: We follow the hint. Since m (a) 0 (mod p), then am 1 (mod p). Hence,
there exist b such that ba 1 mod p (indeed, b = am1 ), so a is relatively prime to p. We prove
that the order of a is m. For a contradiction, suppose ad 1 (mod p) for some d dividing m,
so that d (a) 0 (mod p) for some d < m. Thus, a is a multiple root of xm 1, so is also a
root of its derivative mam1 . Hence, mam1 0 mod p, impossible since p does not divide m
nor a. Therefore, the order of a in (Z/pZ) is precisely m
Exercise 16. Let a Z. Show that if p is an odd prime dividing m (a) then either p
divides m of p 1 (mod m).
Solution: Let p be an odd prime dividing m (a). If p does not divide m, then, by (c), a is
relatively prime to p and the order of a in F
p is m. Since |Fp | = p 1, this implies m divides
p 1, that is, p 1 (mod m).
Exercise 17. Prove there are infinitely many primes p with p 1 (mod m).
Solution: By Exercise 14, there are infinitely many primes dividing m (1), m (2), m (3), . . ..
Since only finitely of them can divide m, then, by Exercise 16, there must exists infinitely many
primes p with p 1 (mod m).

Any comments or suggestions about the solutions are welcome.


positron0802.wordpress.com
positron0802@mail.com

25

Anda mungkin juga menyukai